A&P 3

¡Supera tus tareas y exámenes ahora con Quizwiz!

Which of the following would be a suitable replacement for someone with paralysis of the pharyngeal constrictor muscles and paralysis of muscles that elevate the larynx?

A feeding tube surgically placed between the outside abdominal wall and stomach

Use your worksheet and integrate with what you know about blood pressure controls. Fill in the blanks: When blood pressure or blood volume are too high, ____ will be released and will stimulate the kidney to ____ GFR so that blood volume & pressure will decline.

ANP; increase

Air flow and blood flow are governed by the same principles: flow occurs when pressure gradients can overcome resistance. And like in the cardiovascular system, resistance to airflow is almost entirely altered by changes in diameter of the conduits (in the case of the respiratory system, these important conduits are the bronchioles). What would happen to airflow into an alveolus when the bronchiole serving that alveolus constricts?

Airflow would decrease into the alveolus Right! Downstream of the constriction airflow will decrease. Controlling airflow into alveoli is called alveolar ventilation. Alveolar ventilation is closely matched to blood flow into the pulmonary capillaries (perfusion of the pulmonary capillaries) by a special autoregulatory mechanism. That is, when an alveolus has lots of fresh oxygen and relatively little carbon dioxide, the pulmonary capillary bed (arteriole) serving that alveolus opens up to increase flow (high ventilation is matched by high perfusion). Should the alveolus be full of carbon dioxide and poor in oxygen, the arteriole/capillary restricts blood flow (poor ventilation is met with poor perfusion). This helps increase efficiency in the respiratory system so that we can maximize exchange surfaces.

What would happen to airflow into an alveolus when the bronchiole serving that alveolus constricts?

Airflow would decrease into the alveolus Right! Downstream of the constriction airflow will decrease. Controlling airflow into alveoli is called alveolar ventilation. Alveolar ventilation is closely matched to blood flow into the pulmonary capillaries (perfusion of the pulmonary capillaries) by a special autoregulatory mechanism. That is, when an alveolus has lots of fresh oxygen and relatively little carbon dioxide, the pulmonary capillary bed (arteriole) serving that alveolus opens up to increase flow (high ventilation is matched by high perfusion). Should the alveolus be full of carbon dioxide and poor in oxygen, the arteriole/capillary restricts blood flow (poor ventilation is met with poor perfusion). This helps increase efficiency in the respiratory system so that we can maximize exchange surfaces.

Hemoglobins' affinity for oxygen is directly related to how much oxygen is actually currently held by the hemoglobin. So, if hemoglobin has high oxygen saturation, hemoglobin's affininty for oxygen is high. What do you think happens to hemoglobin's affinity for oxygen after it moves through the systemic capillaries?

As the RBC moves through the systemic capillary, hemoglobin's affinity for oxygen decreases. Correct! Hemoglobin is a protein molecule that behaves differently in different situations. When oxygen level changes, hemoglobin's affinity for oxygen changes. When oxygen is very abundant in the environment (high pO2), hemoglobin binds oxygen very tightly due to the way the molecule is shaped. When environmental oxygen is very low (low pO2), hemoglobin is shaped differently and does not bind oxygen as readily. In the case of the body, when hemoglobin travels through the pulmonary capillary it moves into a very high oxygen environment, causing it to have a high oxygen affinity. When the hemoglobin moves through the systemic capillary, it becomes exposed to a lower oxygen environment and then decreases its affinity for oxygen. This loss of affinity from a high state (bound with lots of oxygen) to a low state (less oxygen bound) causes oxygen to be released to the tissues.

Hemoglobins' affinity for oxygen is how much oxygen is actually currently held by the hemoglobin. So, if hemoglobin has high oxygen saturation, hemoglobin's affininty for oxygen is high. What do you think happens to hemoglobin's affinity for oxygen as it moves through the systemic capillaries?

As the RBC moves through the systemic capillary, hemoglobin's affinity for oxygen decreases. Correct! Hemoglobin is a protein molecule that behaves differently in different situations. When oxygen level changes, hemoglobin's affinity for oxygen changes. When oxygen is very abundant in the environment (high pO2), hemoglobin binds oxygen very tightly due to the way the molecule is shaped. When environmental oxygen is very low (low pO2), hemoglobin is shaped differently and does not bind oxygen as readily. In the case of the body, when hemoglobin travels through the pulmonary capillary it moves into a very high oxygen environment, causing it to have a high oxygen affinity. When the hemoglobin moves through the systemic capillary, it becomes exposed to a lower oxygen environment and then decreases its affinity for oxygen. This loss of affinity from a high state (bound with lots of oxygen) to a low state (less oxygen bound) causes oxygen to be released to the tissues.

How are total cross-sectional of vessels and velocity of blood flow related?

As total cross-sectional area of vessels increases, velocity of blood flow decreases

Using the table above, which leukocyte are you least likely to encounter in a blood smear of a healthy individual?

Basophils

Why should you love your kidney and treat it right?

Because it controls blood pressure Because it determines blood viscosity Because it prevents anemia All of the above Right! The kidney detects low blood oxygen levels and blood pressure. When blood oxygen is too low (anemia), it releases erythropoietin (EPO), a hormone that increases red blood cell production. The more red blood cells produced, the greater the blood viscosity. When blood pressure is too high, the kidney can correct the problem by producing more urine. Urine is really just filtered blood and so with increased urine production, there is less blood volume and therefore less blood pressure. What is very amazing about this pressure/urine volume relationship is that it can work independently of hormones as a simple pressure filter. The higher the pressure, the more urine produced; the more urine made, the lower the blood volume will drop and therefore further reduce blood pressure. This is a life long mechanism for regulating blood pressure. Of course this can work in reverse such that if blood pressure is too low, less urine is made and blood volume with rise (coupled with some fluid intake as well).

Why should you love your kidney and treat it right?

Because it controls blood pressure Because it determines blood viscosity Because it prevents anemia All of the above Right! The kidney detects low blood oxygen levels and blood pressure. When blood oxygen is too low (anemia), it releases erythropoietin (EPO), a hormone that increases red blood cell production. The more red blood cells produced, the greater the blood viscosity. When blood pressure is too high, the kidney can correct the problem by producing more urine. Urine is really just filtered blood and so with increased urine production, there is less blood volume and therefore less blood pressure. What is very amazing about this pressure/urine volume relationship is that it can work independently of hormones as a simple pressure filter. The higher the pressure, the more urine produced; the more urine made, the lower the blood volume will drop and therefore further reduce blood pressure. This is a life long mechanism for regulating blood pressure. Of course this can work in reverse such that if blood pressure is too low, less urine is made and blood volume with rise (coupled with some fluid intake as well).

Why should you love your kidney and treat it right?

Because it controls blood pressure Because it determines blood viscosity Because it prevents anemia Correct! All of the above Right! The kidney detects low blood oxygen levels and blood pressure. When blood oxygen is too low (anemia), it releases erythropoietin (EPO), a hormone that increases red blood cell production. The more red blood cells produced, the greater the blood viscosity. When blood pressure is too high, the kidney can correct the problem by producing more urine. Urine is really just filtered blood and so with increased urine production, there is less blood volume and therefore less blood pressure. What is very amazing about this pressure/urine volume relationship is that it can work independently of hormones as a simple pressure filter. The higher the pressure, the more urine produced; the more urine made, the lower the blood volume will drop and therefore further reduce blood pressure. This is a life long mechanism for regulating blood pressure. Of course this can work in reverse such that if blood pressure is too low, less urine is made and blood volume with rise (coupled with some fluid intake as well).

During heavy exercise, cardiac output increases dramatically, although pressure may only increase modestly. How is this possible?

Because resistance increases at some vessels and decreases at others Right! During exercise, autoregulation to active skeletal muscles causes arterioles serving the muscle capillary beds to dilate and precapillary sphincters to open. Now that blood can find more vessels to fill as arterioles and sphincters open, decreasing resistance and increasing flow to tissues, pressure in the aorta may fall (more active muscle mass would increase number of dilating arterioles and magnitude of pressure drop). Such a drop in pressure would trigger baroreceptor reflexes that would lead to sympathetic activation. Sympathetic activation would trigger vasoconstriction in the systemic vessels except for those serving tissues that are working hard; in such tissues, autoregulation overcomes the sympathetic vasoconstriction and the vessels remain open. The vessels that do constrict (those serving gut tissue), act to increase pressure and shunt blood to other tissues that have decreased resistance. At the same time, the heart works harder (increases HR and contractility due to the sympathetic stimulation) and increases CO. It should seem that the increased CO should increase pressure radically and yet, the balance of the increased CO, vasoconstriction from sympathetic signals and vasodilation from autoregulation results in a high CO, some systemic pressure increase, but little change in the systemic peripheral resistance. In fact, peripheral resistance may even drop overall. Imagine if you needed to increase CO to deliver more blood, but you did not alter resistance (vessels could not change diameter). Pressure would increase dramatically and tear vessels. By altering resistance, we can have huge gains in CO with only a modest pressure increase.

Why can the pulmonary capillary blood achieve the same PO2 as alveolar air?

Because there is an overabundance of oxygen in the alveoli compared to the pulmonary arteries Yes! Air is a mixture of gases. Their total pressure (760 mmHg) is a sum of the pressures exerted by each gas. Partial pressure is the pressure exerted by a single gas inthe mixture of gases. The movement of a gas is related to its individual partial pressure; gases move from a region of higher partial pressure to a region of lower partial pressure, down a partial pressure gradient.

Why do the lungs expand during inspiration?

Because they are "pulled" open by the pleura Right! Increasingly negative intrapleural pressure generated by the expansion of the rib cage and inferior thoracic cavity during inhalation (inspiration) pulls the lungs open. Surface tension between the visceral and parietal pleura create a suction. If the parietal pleura (attached to inside thoracic cavity) move away from the lung, they generate a greater suck on the visceral pleura. The visceral pleura are attached to outside of the lung, and thus, when the visceral pleura moves, so does the lung. No muscles directly insert onto the lungs.

Why are women more prone to bacterial infections of the urinary tract than men?

Because women have shorter urethras Yes! With the much shorter urethras in women, contaminating bacteria from the external world can move into the urinary tract. Inflammation of the urinary tract attempts to combat the bacteria, causing pain. Bacteria can also move into the bladder or even up the ureters to cause kidney infections.

Why is it acceptable to make CO2 in RBCs found in pulmonary capillary blood?

Because you can release CO2 into the alveoli and remove it from the body

On Model 2, when does ventricular systole begin?

Beginning of time period 3

Erik needs a blood transfusion. Which of his friends should donate RBCs to Erik?

Ben bc both have B and Rh, ben also lacks B antibodies

Which of the following make up the highest percentage of whole blood?

Blood plasma and erythrocytes

Suppose your blood has too low pH. How can your body correct this?

By increasing depth and rate of pulmonary ventilation Right! The respiratory system can correct acidosis (too low pH of the blood) by increasing respiratory rate and depth to eliminate more CO2 from the body. Doing this causes more H+ ions not to be free but to combine with HCO3- to make H2CO3 which then makes more CO2 and H20 at the lung. The CO2 is then removed from the system leaving only water. All of the other options would cause more H+ to remain in the body thus making the blood more acidic.

On a moment to moment basis, how do we change vascular resistance and therefore blood flow to our tissues?

By increasing or decreasing vessel diameter. Right! The most important factor affecting vascular resistance is the diameter of the blood vessels. Blood viscosity and vessel length do affect resistance, but those are not factors that change quickly to adjust the amount of blood delivered to rapidly changing tissue demands. Vessel diameter can change quickly but also has a huge effect on resistance because resistance increases as vessel diameter decreases to the fourth power.

What is responsible for pushing fluid out across the capillary wall into the interstitial fluid?

Capillary hydrostatic pressure being greater than interstitial fluid hydrostatic pressure

Look at the equation in Model 2: Inside the RBC, what happens to carbonic anhydrase activity and bicarbonate production if H+ ion concentration rises?

Carbonic anhydrase makes less HCO3-

Which of the following will most readily trigger an increase in respiration rate and depth

Carotid PCO2 = 47 mmHg Right! Femoral venous hemoglobin saturation = 80% is normal, maybe even a little high for a systemic vein according to what we discussed in lecture. Carotid PCO2 = 47 mmHg is too high, this should be =40 mmHg. Very small increases in systemic arterial PCO2 trigger increases in respiration rate to remove CO2 from the body. Pulmonary venous PCO2 = 40 mmHg - this is normal for blood entering the pulmonary capillary for oxygen pickup. Aortic PO2 = 90 mmHg - this is a little low, but hemoglobin is still nearly 100% saturated and this will not alter respiration rate. Jugular PO2 = 40 mmHg - this is normal for blood leaving a tissue at rest.

You have been gardening in a squatting position for 20 minutes. You stand up quickly to answer the phone and feel light headed. After about 10 seconds more you feel fine again. What was the cause and correction of your faint feeling?

Cause: low blood pressure due to too low cardiac output Correction: sympathetic nervous system activation from a baroreceptor signal Right! Squatting for a while reduces venous return from the lower limb because the veins are compressed and cannot refill with blood or send blood toward the heart (try this and look at your feet after 20 minutes of squatting). Reduced venous return will cause reduced cardiac output (Frank-Starling law) and reduced blood pressure (flow = pressure change/R). The cause of your faint feeling was reduced blood flow to the brain due to reduced blood pressure. This reduction in blood pressure associated with posture changes (orthostatic changes) is detected by baroreceptors in the carotid arteries. The correction mechanism to fix the too low blood pressure is vasoconstriction of some peripheral arteries and increased cardiac output, both due to increased sympathetic activation. The information from baroreceptors in the aorta and carotid arteries is relayed to the medulla oblongata cardiovascular centers. When too low pressure is detected, these centers direct increased sympathetic activity to blood vessels and the heart. Increased sympathetic activity increases cardiac output, thus increasing flow in the system and increasing blood pressure overall. The increased sympathetic activation to blood vessels causes increased vasoconstriction of some systemic vessels that leads to an increase in pressure overall (blood vessels to brain escape this vasoconstriction to some degree). These corrective mechanisms can increase systemic blood pressure within 5-10 seconds. Angiotensin II and ANP are chemicals that adjust blood pressure but they do so in a matter of minutes or hours. Angiotensin II production is triggered when blood flow to the kidney is reduced. Angiotensin II causes widespread vasoconstriction and increases in blood volume through decreased urine production (using the hormones aldosterone & ADH). ANP is atrial natriuretic peptide - a hormone released from the heart when pressure is too high. ANP causes increased urine formation - the more urine made, the lower the blood volume will become. Reducing blood volume will reduce blood pressure. The kidney can control blood pressure by adjusting blood volume as a pressure filter. In this manner, the kidney takes hours to days to correct blood pressure, not seconds as indicated here.

What factor determines air flow into or out of the lung? (Assume no factor is 0)

DIfference in pressure between the atmospheric pressure and intrapulmonary pressure

What are the brush border enzymes?

Enzymes that are in the cell membranes of small intestine cells Right! The brush border enzymes are the enzymes that are locked into the cell membranes of the microvilli of the cells of the small intestine. They typically are responsible for the last stage of chemical digestion that breaks dietary stuffs into the most basic building blocks (monomers) for immediate absorption. They usually are not freely floating in the chyme. The freely floating enzymes are made by the pancreas and released into the duodenum where they can begin their work of chemical digestion. The liver cells make bile, but not enzymes. Bile is not an enzyme but instead a mixture of salts that breaks fats into smaller pieces (emulsifies fat). These smaller pieces of fat have more surface area for enzymes to act on. Fat digesting enzymes, called lipases are made in small amounts by the tongue, salivary glands and stomach. In larger amounts, the pancreatic lipases work on fats emulsified by bile. The stomach makes pepsin, a protein digesting enzyme. This enzyme is released into the lumen of the stomach and freely floats around digesting proteins it finds.

True or False? The large intestine is responsible for the absorption of most of the water in the GI tract.

False Correct. Water absorption in the GI tract, like everywhere else, is a passive process that occurs when ions/solutes are absorbed. Since most solutes (nutrients) are absorbed in the small intestine, water follows, accounting for most water absorption in the GI tract also in the small intestine.

How do cells of the liver (hepatocytes) receive oxygenated blood?

From hepatic artery Right! The liver receives oxygenated blood from the hepatic arteries. The hepatic portal vein brings nutrients that were absorbed in the intestines to the liver for removal and repackaging. Once the nutrients have been removed from the blood, appropriate nutrients re-enter the blood and circulate to the tissues. If the body does not need certain nutrients, the liver converts them to storage molecules or incorporates them into bile. Bile ducts contain bile and the central vein drains blood from the liver to the inferior vena cava.

Compare this image to the baseline in Model 1. How would GFR change in this image relative to baseline?

GFR would increase

Compare this image to the baseline in Model 1. How would glomerular hydrostatic pressure change in this image relative to baseline?

Glomerular hydrostatic pressure would increase

According to Model 3, what does hemoglobin release when it binds O2?

H+

What does hemoglobin release when it binds O2?

H+

What happens to hemoglobin's affinity for oxygen when levels of CO2 increase?

Hemoglobin's affinity for oxygen decreases Nice! When the level of CO2 increases (or conditions become acidic or temperature increases), hemoglobin's affinity for oxygen decreases. This means that hemoglobin holds on to oxygen less tightly, thus releasing it more readily to a hungry tissue. This is also a right shift in the oxygen-hemoglobin dissociation curse and is beneficial because it allows more oxygen delivery for the same level of PO2. It would seem that this lower affinity of hemoglobin for oxygen would be bad because it would limit how much O2 could bind in the lung. But remember that hemoglobin is fully saturated at low levels of PO2 naturally (as low at 70 mmHg) and the high level of CO2 is erased at the pulmonary capillary when CO2 diffuses into the alveoli and is removed from the blood. So, while the hemoglobin curve shifts right when the hemoglobin is in the systemic capillaries, it shifts to the left again when the hemoglobin is in the pulmonary capillaries.

What happens to hemoglobin's affinity for oxygen when blood becomes more acidic?

Hemoglobin's affinity for oxygen decreases Nice! When the level of CO2 increases (or conditions become acidic or temperature increases), hemoglobin's affinity for oxygen decreases. This means that hemoglobin holds on to oxygen less tightly, thus releasing it more readily to a hungry tissue. This is also a right shift in the oxygen-hemoglobin dissociation curse and is beneficial because it allows more oxygen delivery for the same level of PO2. It would seem that this lower affinity of hemoglobin for oxygen would be bad because it would limit how much O2 could bind in the lung. But remember that hemoglobin is fully saturated at low levels of PO2 naturally (as low at 70 mmHg) and the high level of CO2 or H+ is erased at the pulmonary capillary when CO2 diffuses into the alveoli and is removed from the blood. So, while the hemoglobin curve shifts right when the hemoglobin is in the systemic capillaries, it shifts to the left again when the hemoglobin is in the pulmonary capillaries.

Which of the following will INCREASE respiratory rate?

High carotid PCO2 If there is too much CO2 in the systemic arterial blood, then there was not enough removal of CO2 by the lungs. The chemoreceptors in the carotid bodies & aorta detect this high systemic arterial PCO2 and relay the information to the respiratory centers of the brainstem. Also, central chemoreceptors located in the medulla oblongata receive CO2 from systemic arteries. If too much CO2 moves across into the blood brain barrier, H+ ions form. These ions cannot be buffered (due to low protein composition of the cerebrospinal fluid) and the central chemoreceptors are triggered. The brainstem centers (in pons and medulla oblongata) are then stimulated to increase respiratory rate to remove more CO2 from the body. We do not monitor the composition of the systemic venous blood (i.e. Jugular vein) because the blood in these vessels has not reached the lungs for adjustment. Not matter how high the systemic venous blood CO2 level becomes, If the lungs are performing adequately, then enough CO2 is removed and there is no cause to increase ventilation rate. Too low O2 can stimulate increased respiration rate, but only when O2 levels reach less than 60 mmHg PO2 in systemic arteries.

A patient arrives at your emergency department (ED) feeling very weak and faint after spending the day drinking alcohol in the sun at a baseball game. You measure her radial blood pressure as 80/55 mmHg. You diagnose severe dehydration and give her intravenous (IV) fluids slightly hypotonic to her blood. For your patient, before treatment with IV fluids, how does her lymph flow compare to normal?

Higher than normal

Based on your understanding of capillary dynamics, which of the following would you expect will increase glomerular filtration rate (fluid leaving the first capillary in the nephron)?

Increased blood plasma volume

Which of the following accurately describes the site of external respiration?

It allows easy diffusion of gases between the alveoli and the body. Right! The respiratory membrane is the site of external respiration. It is made of the pulmonary capillary wall, a thin amount of connective tissue and the wall of an alveolus. It is free of cilia although it does have a little bit of fluid lining it. The respiratory membrane is very thin to allow easy diffusion of gases - in pneumonia the respiratory membrane becomes thickened and it is difficult to load the blood with oxygen because gases cannot diffuse into the blood.

In Model 3, in the systemic capillaries, what happens to the H+ ion created by the conversion of CO2 (+H2O) to HCO3?

It becomes attached to hemoglobin, releasing an O2 to the tissue

In Model 3, in the pulmonary capillary, what happens to the H+ ion when O2 binds with hemoglobin?

It binds with HCO3- to make CO2 (& H2O)

The respiratory membrane is found at the site of external respiration. What best describes the respiratory membrane?

It exists between the walls of the alveoli and the walls of the pulmonary capillaries Right! The respiratory membrane is the site of gas exchange - the site of external respiration. It is a very thin barrier made of the walls of the alveoli (type I alveolar cells) and the endothelial cells of the pulmonary capillary. Because it is so thin, it easily allows the diffusion of gases between the alveoli and the capillary blood. The respiratory epithelium lines the nasal cavity, trachea and bronchioles - proximally it is PCCE and serves to cleanse, humidify and warm incoming air. The intrapleural space is between the visceral and parietal pleura; the pleura properly are created by a simple squamous epithelium and underlying connective tissue. Although anatomically similar to half of the respiratory membrane, the pleura produce a fluid that creates surface tension and lubricates movements of the lungs.

Which of the following best describes the blood in the pulmonary veins?

It has a PO2 of ~100 mmHg Right! Pulmonary arteries bring deoxygenated blood from the right ventricle to the pulmonary capillaries. In the pulmonary capillaries the blood picks up oxygen from the alveoli (and drops off carbon dioxide). Oxygenated blood then returns to the left atrium via the pulmonary veins. Whatever the conditions are in the alveoli in terms of partial pressures of oxygen or carbon dioxide will become the condition of the pulmonary capillary, and thus the pulmonary venous blood. When blood enters the pulmonary capillaries, exchange can occur between the alveoli and the blood. Because the alveoli have much more gas than the blood can ever hold, the capillary blood becomes equivalent to the alveoli in terms of gas partial pressures. Please see the videos posted in Multimedia folder in reference to gas exchange & partial pressures.

Who's blood contains Anti-A antibodies?

James & Maggie NOT agglutinated anti A

Which of the following occur when ATP production increases at the tissues?

More CO2 is produced in the RBCs at the pulmonary capillary Right! When the tissues begin making more ATP they also make more CO2 (by aerobic metabolism). That CO2 diffuses into the blood and causes more O2 to be offloaded because high CO2 decreases hemoglobin's affinity for O2 and thus more O2 is released to the tissues. More H+ ions are made and the hemoglobin can buffer them because they have given up more O2 and now can bind more H+ to form HHb. At the tissues, more HCO3- is made when CO2 production increases. The confusing part is that the HCO3- and HHb travel in the blood up to the lungs and in the pulmonary capillary, the H+ comes off of the HHb (as oxygen binds to Hb) and binds to the HCO3-. When this happens, CO2 and H2O are formed. SO, CO2 is made in RBC in pulmonary capillaries because that CO2 can then diffuse into the alveoli and be removed from the body. Albumin does not transport CO2.

Which layer of the GI tract participates in chemical digestion and absorption?

Mucosa

Which of the following would you expect to have the thickest tunica media relative to their overall diameter?

Muscular arteries Right! Muscular arteries have large amounts of smooth muscle in their tunica media. Sympathetic nerves innervate this muscle and when active determine the vessel diameter and thus blood pressure and blood flow. They are important in maintaining vascular tone. Venules are thin walled and porous; veins are not porous but are thin walled. Arterioles also have some smooth muscle and are important in their location immediately before the capillary. They regulate blood flow into the capillary and thus the amount of blood available to the tissues.

Where does most reabsorption of the filtrate occur (most by total volume of the filtrate)?

PCT Yes! Most of the filtrate is reabsorbed immediately in the proximal convoluted tubule. The remaining filtrate is reabsorbed along the Loop of Henle, DCT & collecting duct. The hormones ADH & aldosterone affect the amount of water and salt reabsorbed in the DCT & collecting duct. Even though most of the filtrate is reabsorbed by the time it reaches the DCT & collecting duct, the affect of ADH is significant enough to allow more water to be reclaimed and a very low volume, concentrated urine can be produced.

Which of the following will cause fluid to collect in the interstitial space (create tissue edema)?

Permeability of the lymphatic capillaries decreases Right! If the lymphatics cannot drain away the fluid in the interstitial space, fluid accumulates - this condition is called edema. The more permeable the lymphatics, the more fluid it can drain. If the blood capillary becomes less permeable or blood pressure drops, less fluid leaves the systemic capillaries to create interstitial fluid. If blood albumin level increases, more fluid will be kept in the capillary (more pulled in to oppose capillary hydrostatic pressure) and less fluid will enter the interstitial space. Administration of proteins in IV fluid therapy is a way to increase blood volume and maintain that blood volume because the proteins act to suck fluid towards them so that less fluid can become interstitial fluid.

What is the correct path of a red blood cell as it moves through the healthy kidney?

Renal artery - afferent arteriole - glomerulus - efferent arteriole - peritubular capillary - renal vein - inferior vena cava Yes! Blood cells are not filtered into the Bowman's capsule. They remain within the walls of the blood vessels and so this question really just asks about the flow of blood through the kidney. This answer would be the same if the question asked What is the correct order for a sodium ion that is not filtered at the glomerulus or secreted into the tubule? Please review blood flow through the kidney using your lecture notes so that you know what vessels you are expected to know.

How is secretion different from reabsorption in the kidney?

Secretion and reabsorption both use membrane proteins for substance transport, but secretion moves substances from the blood into the filtrate and reabsorption moves substances from the filtrate into the blood. Right! Filtration creates filtrate - it is the process where the blood pushes plasma like filtrate out of the glomerulus into the Bowman's capsule (also called glomerular capsule). It is passive and requires no ATP. Filtration allows anything dissolved in the plasma to leave the blood, except proteins and large molecules like antibiotics. It does not allow blood cells to leave the blood vessel. Reabsorption uses active processes (either directly or indirectly) to remove filtered substances from the filtrate to the blood. Most of reabsorption will occur in the proximal convoluted tubule (PCT) using sodium ion transport proteins that require ATP. Other molecules will then be linked to the Na+ movement (glucose, amino acids, HCO3- ions etc) and water will follow. Secretion is reabsorption in reverse - it uses membrane proteins to move substances from the peritubular capillary through the tubule cells into the filtrate. It is very useful if the blood contains excess solutes or the molecule was too big to be filtered at the glomerulus into the original filtrate. It can occur in the PCT or the distal convoluted tubule (DCT).

Why must GFR be kept at a constant level?

So that you filter your blood to remove wastes So that you can maintain blood pressure So that you can maintain the proper ion concentration in your blood All of the above Right! A balanced rate of glomerular filtration ensures that your blood gets filtered at a rate which your nephron tubules can work with the filtrate inside of them. If the GFR is too high, the filtrate passes through the nephron too fast and solutes (& water) do not get reclaimed (reabsorbed). If GFR is too low, too much volume get reabsorbed and waste products do not leave the body. Maintaining blood composition and volume factor into the maintenance of blood pressure. You must maintain an appropriate rate of glomerular filtration to constantly adjust your blood levels of ions and wastes. If you do not filter your blood, body pH is not maintained and ionic composition changes. These in turn affect cellular functions such that neurons may not fire or the heart may not beat. Enzymes in the body also need a steady environment in which to work.

Total cross-sectional area refers a group of vessels (i.e. all the systemic capillaries). If we were to cut into every systemic capillary and measure their cross-sectional areas then sum these together, we would get total cross-sectional area for the systemic capillaries. Using the figure above, which vessels have the largest total cross-sectional area?

Systemic capillaries

Using the figure above, which vessels have the smallest diameter?

Systemic capillaries

Using the above figure, which vessels have the lowest average blood pressure?

Systemic veins

Where would you find hemoglobin that is relatively O2 poor in a healthy resting person?

Systemic veins & pulmonary arteries Great! During rest, the tissues consume O2 to make ATP. Blood that exchanges respiratory gases with the air in the alveoli picks up oxygen to become fully loaded with O2 (hemoglobin saturated to maximum, effectively 100%). This O2 rich blood leaves the pulmonary capillaries and flows to the pulmonary veins, left atrium, left ventricle and then the systemic arteries. When systemic arterial blood arrives at the resting tissues, the blood releases some of the O2 for the tissue cells to use. The blood then leaving the tissues will have less O2 (depending on how much was released to the tissue cells) and so systemic venous blood is relatively O2 poor (at rest, it has 75% of the O2 that they systemic arterial blood had). This systemic venous blood moves into the right atrium, right ventricle and then the pulmonary arteries. Because gas exchange in/out of the blood only occurs in 2 places (the pulmonary capillaries or the systemic capillaries) the amount of O2 in the systemic venous blood is equal to the amount of O2 in the pulmonary arteries.

Where would you find hemoglobin that is 75% saturated with O2 in a healthy resting person?

Systemic veins & pulmonary arteries Great! During rest, tissue PO2 is 40 mmHg. Blood that loads oxygen in the pulmonary capillaries becomes fully saturated with O2 (hemoglobin saturated to maximum, effectively 100%). When blood arrives at the resting tissues and equilibrates, the blood achieves a PO2 of 40 mmHg. At 40 mmHg, hemoglobin is only 75% saturated (as determined by the oxygen-hemoglobin dissociation curve). Therefore, systemic venous blood is 75% saturated with oxygen. O2 does not leave the blood except at capillaries and so as blood flows through the heart from the systemic veins to the pulmonary arteries, no further oxygen is lost. Acordingly, pulmonary arterial blood has a PO2 =40 mmHg and also contains hemoglobin that is 75% saturated with oxygen.

In normal, healthy individuals, the diaphragm moves superiorly during exhalation.

TRUE Yes! When the diaphragm contracts, it moves inferiorly, increasing the size of the thoracic cavity to allow air to flow into the lungs. During quiet exhalation, the diaphragm relaxes and passively moves superiorly to decrease the size of the thoracic cavity and force air out of the lungs.

What happens in an otherwise normal individual if their liver cannot produce albumin?

The blood would lose fluid to the tissues and the tissues would become swollen (edema)

What would happen if intrapleural pressure became 0 mmHg when thoracic volume increased?

The lungs would not expand Right! Negative intrapleural pressure creates a suction between the lung and chest wall (thoracic body wall). The parietal pleura line the inside of the chest wall; the visceral pleura are the outermost layer of the lung. Between the two layers is some fluid that creates surface tension and a sucking pressure (negative pressure). This negative pressure keeps the lungs sucked against the chest wall. Because the chest wall has a greater volume than the lungs and a volume that cannot drop below the limits of the bony ribs, the lungs are always somewhat inflated (open). When the chest wall expands during inhalation, the intrapleural pressure becomes more negative (goes from -4 mmHg to -8 mmHg) thus pulling the lungs outward further (inflating the lung). If this pressure were to become 0 mmHg (meaning equal to atmospheric pressure), there would be no suck between the pleura and the lung would collapse due to its own elastic forces.

What is the most important force causing interstitial fluid to return to the blood at the venous end of the systemic capillary?

The presence of proteins in the blood Right! The presence of proteins in the blood determines the osmotic pressure of the blood. It acts to suck fluid into the capillary. At the venous end, it is greater than the blood pressure (capillary hydrostatic pressure) that pushes fluid out to the tissue cells (interstitial space). Therefore, the osmotic suck overcomes the blood hydrostatic pressure push and fluid moves into the capillary. Interstitial fluid hydrostatic pressure is the pressure generated by the fluid in the tissue. It pushes fluid back into the capillary, although there is not much of it and it is therefore not high. Interstitial fluid osmotic pressure is created by proteins in the fluid. There are not many (if any) proteins in the interstitial fluid and therefore this is not high. Even if it were high, it would tend to pull (suck) fluid out of the capillary and not push it back in. High blood hydrostatic pressure - this is blood pressure and it is not high at the venous end. Even if it were, it would tend to push fluid out of the capillary. Low blood colloid osmotic pressure - this is the pressure sucking fluid back into the capillary caused by the presence of proteins in the blood. The lower it is, the less the suck and really, it is not any lower at the venous end. If it were, more fluid would stay in the interstitial space

Where does most reabsorption of the filtrate occur (most by total volume of the filtrate)?

The proximal convoluted tubule Yes! Most of the filtrate is reabsorbed immediately in the proximal convoluted tubule. The remaining filtrate is reabsorbed along the Loop of Henle, DCT & collecting duct. The hormones ADH & aldosterone affect the amount of water and salt reabsorbed in the DCT & collecting duct. Even though most of the filtrate is reabsorbed by the time it reaches the DCT & collecting duct, the affect of ADH is significant enough to allow more water to be reclaimed and a very low volume, concentrated urine can be produced.

Which of the following is important to establishing the concentration gradient observed in interstitial fluid of the renal medulla?

The variable permeability of the loop of Henle Right! The ascending thick limb of the loop of Henle contains salt pumps and is impermeable to water. The descending limb is permeable to water. When salt is pumped out on the ascending side, that salt moves into the interstitial space. This increases the saltiness in the fluid (interstitial fluid) around the descending limb and draws water out of the descending limb (the water leaves the filtrate to try to dilute the interstitial fluid). As water leaves the descending limb, the filtrate becomes saltier. Then this saltier filtrate passes through the hairpin turn of the loop and up into the ascending limb. In the ascending limb salt pumps remove even more salt out to interstitial fluid around the descending limb. The more times this happens, the more salty the interstitial fluid becomes (this is the countercurrent multiplier). Also, because more salt is pumped toward the bottom of the ascending limb, the interstitial fluid of the deeper medulla becomes saltier than the superficial medulla. Angiotensin II and aldosterone do not affect this process. A very high GFR would result in less time for salt pumping and would eliminate (or diminish) the medullary concentration gradient.

A patient arrives at your emergency department (ED) feeling very weak and faint after spending the day drinking alcohol in the sun at a baseball game. You measure her radial blood pressure as 80/55 mmHg. You diagnose severe dehydration and give her intravenous (IV) fluids slightly hypotonic to her blood. For your patient, what is happening in her heart when her systemic arterial blood pressure is 80/55 mmHg?

To open the aortic valve, her heart is generating at least 55 mmHg pressure

In Model 3, which net direction does the CO2 equation move when the RBC is in the systemic capillary?

To the right (to produce H+ & HCO3-)

What happens if the interstitial fluid and intracellular fluids are not (on average) iso-osmotic?

Too much water either enters or leaves the tissue cells causing them to swell or dehydrate

The lower hemoglobin's affinity for oxygen, the more hemoglobin can prevent blood acidosis.

True Right!! Affinity here refers to hemoglobin's greediness for oxygen. When affinity is high, hemoglobin binds oxygen tightly and does not release it. When affinity is low, hemoglobin releases oxygen readily. High levels of CO2, H+ ions (low pH) and high temperature cause shape changes in hemoglobin molecules that decrease hemoglobin's affinity for oxygen. When hemoglobin releases an oxygen molecule, it provides a binding site on the hemoglobin for an H+. Free H+ cause pH to decline - creating acidosis - but H+ bound to a protein cannot affect pH (the binding protein buffers against pH changes). Hemoglobin does just this - it binds a H+ when oxygen affinity is low and acts to prevent acidosis of the body.

In normal, healthy individuals, the diaphragm moves superiorly during exhalation.

True Yes! When the diaphragm contracts, it moves inferiorly, increasing the size of the thoracic cavity to allow air to flow into the lungs. During quiet exhalation, the diaphragm relaxes and passively moves superiorly to decrease the size of the thoracic cavity and force air out of the lungs.

Use this graph for the following questions. This is a graph of:

an action potential of a contractile cardiac muscle cell.

A powerful vasoconstrictor activated Renin

angiotensin II

bile:

breaks large fat globules into smaller ones. Right! Bile salts break large fat globules into smaller ones so that fat digesting enzymes (lipases) can have more surface area to work on. They are necessary for proper fat digestion. Vitamin B12 requires intrinsic factor from the stomach to aid in absorption. Bile is made in the liver and stored in the gall bladder.

bile

breaks large fat globules into smaller ones. Right! Bile salts break large fat globules into smaller ones so that fat digesting enzymes (lipases from the pancreas) can have more surface area to work on. They are necessary for proper fat digestion. Vitamin B12 requires intrinsic factor from the stomach to aid in absorption. Bile is made in the liver and stored in the gall bladder. It contains no digestive enzymes.

As you move down the bronchial tree (from the trachea toward the alveoli), what changes do you observe.

cartilage decreases

Although the heart is an intermittant pump, _______ expand and recoil thus exerting continuous pressure on the blood and maintaining constant blood flow in the CV system.

elastic arteries Right! The large amounts of elastic tissue in the tunica media of elastic arteries (like the aorta) allow these vessels to expand and recoil to accept the blood ejected from the ventricles. When they expand, they store energy in their fibers that they impart to the blood as they recoil. The vessels recoil during ventricular non-ejection (all of diastole and isovolumetric contraction of systole).

Activation of circular and longitudinal smooth muscle of the GI tract requires a neural signal from the central nervous system.

false

The uvula prevents food and liquid from entering the larynx.

false

True or False? Sympathetic stimulation increases heart rate but decreases stroke volume due to less time for ventricular filling.

false

True or False? The large intestine is responsible for the absorption of most of the water in the GI tract.

false Correct. Water absorption in the GI tract, like everywhere else, is a passive process that occurs when ions/solutes are absorbed. Since most solutes (nutrients) are absorbed in the small intestine, water follows, accounting for most water absorption in the GI tract also in the small intestine. Going for 100% on this one!

Which of the following is released from enteroendocrine cells (G-cells) of the stomach?

gastrin

How do cells of the liver (hepatocytes) receive oxygenated blood?

hepatic arteries Right! The liver receives oxygenated blood from the hepatic arteries. The hepatic portal vein brings nutrients that were absorbed in the intestines to the liver for removal and repackaging. Once the nutrients have been removed from the blood, appropriate nutrients re-enter the blood and circulate to the tissues. If the body does not need certain nutrients, the liver converts them to storage molecules or incorporates them into bile. Bile ducts contain bile and the central vein drains blood from the liver to the inferior vena cava.

In the pulmonary capillaries

hydrogen ions combine with bicarbonate ions to form carbon dioxide and water Right! In the pulmonary capillaries oxygen loads into the blood and CO2 moves into the alveoli. CO2 is made in the systemic tissue mitochondria (there are no mitochondria in the RBCs) and transported away from tissue cells using the blood. Some CO2 dissolves in the plasma, some attaches to hemoglobin (NOT at the same binding site for oxygen) and the rest is converted to bicarbonate ions. The most conversion occurs inside RBCs where the enzyme carbonic anhydrase combines CO2 & water to create a hydrogen ion (H+) and bicarbonate ion (HCO3-). [Even though all the components of CO2 are found in bicarbonate ion, they are in the form of bicarbonate ion and not CO2.] The RBC then pushes bicarbonate out to the blood plasma and hemoglobin binds the H+. Some dissolved CO2 spontaneously combines with water to also make bicarbonate and H+ - acidifying the venous blood. The RBC then moves through the vascular tree up to the pulmonary capillaries. Once in the pulmonary capillaries, CO2 must move out of the blood into the alveoli for removal from the body. Bicarbonate ion cannot diffuse out of the blood into the alveolus, instead we need to "remake" CO2. To recreate CO2, bicarbonate ions combine with H+ and form CO2 and water in the RBC (also due to carbonic anhydrase). The CO2 then diffuses into the alveoli. The blood leaving the pulmonary capillary has less CO2 dissolved (because dissolved CO2 also moved into the alveoli) and therefore also has less free H+, making the pulmonary venous blood less acidic than the pulmonary arterial blood. O2 does not combine with water to make bicarbonate.

Basophil

inflammatory response, often to allergies

Using Model 3, why does increasing blood plasma protein concentration impact GFR the way it does?

it increases glomerular colloid osmotic pressure, thus decreasing GFR

Maintains blood pressure by regulating blood volume

kidney

Where is bile made?

liver Right! Bile is made in the liver by hepatocytes. It is stored in the gall bladder and released at the appropriate time into the small intestine.

During reabsorption, solutes and water move out of the renal tubule into the interstitial space. Most of these reabsorbed materials then ____.

move into the peritubular capillary by diffusion or osmotic pull. Right! During reabsorption, molecules first must leave the tubule lumen, then cross into the interstitial space. From there, reabsorbed substances move into the peritubular capillaries (or vasa recta). These molecules are pulled into the blood supply because the capillaries contain dissolved proteins that create an osmotic pull for the water, pulling in any dissolved substances as well. Some molecules enter the capillary because they move down their own diffusion gradients. The slow speed of the blood moving through the capillaries is very important here to allow for the entry of reabsorbed substances. While lymphatics are present in the kidney, most reabsorbed substances do not return to the circulation through the lymphatics. If they did, the renal lymphatics would be responsible for moving about 180 L/day! If the majority of the reabsorbed substances remained in the kidney, the kidney volume would become enormous, but also, the blood would not maintain proper water, solute and waste balance. Since the blood supplies fluid to tissues, the tissue fluid would then become unbalanced and tissue cells would not be able to perform their duties. Any fluid entering the renal pelvis is to be lost to the environment as urine. Reabsorbed substances would not be in the urine (unless they were later secreted after being reabsorbed, and even still, this is not most of the substances).

During reabsorption, solutes and water move out of the renal tubule into the interstitial space. Most of these reabsorbed materials then ____.

move into the peritubular capillary. Right! During reabsorption, molecules first must leave the tubule lumen, then cross into the interstitial space. From there, reabsorbed substances move into the peritubular capillaries (or vasa recta). These molecules are pulled into the blood supply because the capillaries contain dissolved proteins that create an osmotic pull for the water, pulling in any dissolved substances as well. Some molecules enter the capillary because they move down their own diffusion gradients. The slow speed of the blood moving through the capillaries is very important here to allow for the entry of reabsorbed substances. While lymphatics are present in the kidney, most reabsorbed substances do not return to the circulation through the lymphatics. If they did, the renal lymphatics would be responsible for moving about 180 L/day! If the majority of the reabsorbed substances remained in the kidney, the kidney volume would become enormous, but also, the blood would not maintain proper volume, or water, solute and waste balance. Since the blood supplies fluid to all tissues, the tissue fluids would then become unbalanced and tissue cells would not be able to perform their duties. Any fluid entering the renal pelvis is to be lost to the environment as urine. Reabsorbed substances would not be in the urine (unless they were later secreted after being reabsorbed, and even still, this is not most of the substances).

Which layer of the GI tract participates in chemical digestion and absorption?

mucosa

Which layer of the GI tract participates in mechanical digestion?

muscularis externa

Vasodilation in response to decreased local blood pressure

myogenic autoregulation

respiratory anatomy

nasal cavity, pharynx, larynx, trachea, bronchus, bronchioles

Absence of ____ results in the medical condition pernicious anemia, which is a low oxygen carry capacity of the blood.

parietal cells

lung layer anatomy

parietal pleura, pleural cavity, visceral pleura, lung

Which type of motion dominates in the esophagus?

peristalsis

Which of the following will result in the least amount of gastric gland secretion?

presence of fatty/protein rich chyme in the duodenum Presence of fatty/protein rich chyme in the duodenum triggers the intestinal phase of gastric regulation and greatly reduces gastric secretions

Which type of motion dominates in the intestines?

segmentation

Using this image (Model 3 from worksheet), what factor decreases GFR the most?

severe blood loss

What contributes the most to the osmotic pressure driving water movement towards the interstitial fluid?

sodium ions

The ____ return blood from the systemic circulation into the ____

superior and inferior vena cavae; right atrium

Corrects a sudden drop in blood pressure within seconds

sympathetic activation

Which of the following will be the response of the nephron to increase filtrate formation in response to too little filtrate flowing past the distal convoluted tubule?

the afferent arteriole will dilate

Which of the following will NOT occur during the gastric phase of digestive secretion?

the small intestine is maximally active Correct! During the gastric phase of digestive secretion, the stomach is maximally active. Distention of the stomach triggers reflexes that activate mucosal cells to release secretions and stomach muscle contractions that mix the contents of the stomach. The hormone gastrin will also be released from stomach enteroendocrine cells of the mucosa; gastrin will travel into the blood and cause the ileocecal valve to open and movements of the large intestine that propel food to the rectum. These actions will allow for progression within the GI tract, making room for the new material. When the stomach is maximally active, the small intestine is resting, awaiting chyme.

True or False? Aldosterone causes more sodium reabsorption from the kidney into the blood but does not change the overall osmolarity of the blood.

true

True or False? Following clotting, actin and myosin fibers in platelets help draw the edges of the wound together.

true

True or False? Swallowing has both voluntary and involuntary components.

true

True or False? During the intestinal phase of digestive secretion, hormones released from the small intestine inhibit the action of the stomach.

true Correct! When chyme moves into the small intestine, the hormones secretin CCK are released and neurons are activated. The hormones travel in the blood to reach their receptors on the stomach, liver, gall bladder and pancreas. Although secretin CCK increase activity in the gallbladder, liver (to release bile) pancreas (to release bicarb juice and enzymes), they stimulate relaxation in the stomach and inhibit enzyme release in the stomach. Once the chyme in the small intestine is absorbed or moved along, less secretin CCK are released and the stomach can once again release acid, enzymes and increase activity (assuming there is still food in stomach).

Think about the equation: HHb + O2 --> HbO2 + H+ when answering: The lower hemoglobin's affinity for oxygen, the more hemoglobin can prevent blood acidosis.

true Right!! Affinity here refers to hemoglobin's greediness for oxygen. When affinity is high, hemoglobin binds oxygen tightly and does not release it. When affinity is low, hemoglobin releases oxygen readily. High levels of CO2, H+ ions (low pH) and high temperature cause shape changes in hemoglobin molecules that decrease hemoglobin's affinity for oxygen. When hemoglobin releases an oxygen molecule, it provides a binding site on the hemoglobin for an H+. Free H+ cause pH to decline - creating acidosis - but H+ bound to a protein cannot affect pH (the binding protein buffers against pH changes). Hemoglobin does just this - it binds a H+ when oxygen affinity is low and acts to prevent acidosis of the body.

Which of the following prevent backflow in the large veins of the lower extremity?

venous valves Right! Just like in the heart, valves in the veins prevent backflow of blood (backflow back down to the feet in the case of the veins). Skeletal muscle contractions also help venous blood move upward toward the heart, as does negative intrathoracic pressure generated during ventilation of the lungs. The vasa vasorum is the blood supply of the vessels - the vessels are tissue too and need their own source of nutrients to live. The blood moving though vessels cannot supply all the vessel tissue with nutrients, just like the blood inside the heart chambers does not supply nutrients to the heart muscle.

When you think about food:

your stomach increases motility

What vessel would have the lowest flow?

A small diameter (internal diameter = 1 mm) vessel with a proximal-distal change in pressure of 25 mmHg

What does the EKG (ECG) measure?

All the electrical activity of the heart during a heart beat

After returning from her spring break trip to India, Ashley returns to find that she is feeling run down and feverish. He blood work reveals a higher than normal lymphocyte count, but her blood is otherwise unremarkable. What medication is likely prescribed to Ashley?

Anti-viral

Erythrocytes:

Are composed mainly of intracellular gas transport proteins

Why is resting heart rate lower than the automatic depolarization rate of the SA node?

At rest, the vagus nerve causes SA node cells to hyperpolarize Right! The Vagus nerves synapse on the SA node and AV node. They release acetylcholine (Ach) that binds to chemically gated channels on these cells. The Ach triggers potassium release from the SA & AV node cells, causing the SA node to reach threshold less often and fire action potentials less often (a slower heart rate). Sympathetic activation increases heart rate by allowing calcium entry into the SA node cells (increases contractility too through the same mechanism). Venous return is low at rest. The Bainbridge reflex is triggered when increased venous return activates stretch receptors in the right atrium. These receptors communicate with the cardioacceleratory center in the brainstem which then activates sympathetic nerves to increase heart rate. Although this reflex is activated during inhalation when thoracic pressure increases blood flow into the right atrium, it would not lower the heart rate and it is not dominant.

What does the P wave of the EKG show?

Atrial depolarization

Who will never form antibodies against Rh antigens?

Ben, Erik & Vanessa Rh+ coag antiRH, will never form antibodies against Rh antigens

Valves ensure unidirectional flow through the cardiovascular system. Which of the following structures prevents inappropriate blood flow backward from the left ventricle into the left atrium?

Bicuspid (left AV) valve Right! Blood should flow from the atria into the ventricles and not in the reverse. Pressure is the driving force for blood flow. When ventricular pressure (during ventricular contraction, also called ventricular systole) exceeds atrial pressure, the blood would try to flow from high pressure in the ventricles into the lower pressure atria. The AV valves prevent this action. The same mechanisms apply to prevent backward blood flow from the great vessels (pulmonary trunk and aorta) into the ventricles. After blood ejection from the ventricles, the ventricles begin to relax (relaxation = diastole). As they relax, their pressure drops below that of the great vessels. The aortic and pulmonary semi-lunar valves close as ventricular pressure drops below arterial pressure so that blood cannot flow backward into the ventricles. If a valve does not close properly to prevent backflow, the valve is said to be incompetent. Physicians can hear some improperly closing valves using a stethoscope - when they hear inappropriate heart sounds, it is called a heart murmur. Murmurs are distinguishable from normal heart sounds because they sound like a "whoosh" instead of the "Lubb" or "Dupp" of normal heart function during a heart beat. Check out the heart sounds link in Multimedia Folder of Course Docs for sounds and images of murmurs.

Which of the following is the most accurate description of whole blood with a hematocrit of 60%?

By volume, it is composed mostly of red blood cells. Right! A hematocrit of 60% means that by volume, a sample of whole blood is 60% red blood cells and ~40% plamsa. White blood cells (leukocytes) and platelets should also never be so abundant that they make up 60% of the whole blood volume. This condition of having too high RBCs (erythrocytes) is called polycythemia. This can result from many causes, including blood doping

Which of the following is NOT part of the localized response to damage of a small blood vessel?

Cardiac output (blood flow) increases

During exercise, what happens to the heart?

Cardiac output increases to increase blood delivery to exercising tissues. Right! During exercise there is increased demand at the skeletal muscles for oxygen and nutrients. Receptors detecting muscle activation (proprioceptors) as well as changing levels of blood oxygen and carbon dioxide (reflecting increased usage/production at the tissues) cause the heart to pump more blood each minute. This is an increase in cardiac output to match blood delivery to tissue demands. Also, exercising muscles push more blood through the systemic veins - this is called an increase in venous return. This increased volume into the heart (increased venous return) also leads to increased SV out of the heart (due to the Frank-Starling law of the heart) and thus increases cardiac output. Cardiac output must increase during exercise. Any response that claims that cardiac output decreases is wrong here. While HR does increase during exercise, and this does allow less time for ventricular filling, there is also an increase in heart contractility associated with exercise (due to sympathetic activation). This causes SV to remain the same (or perhaps become even higher as the heart squeezes harder and ESV declines) and therefore the combination of increased HR & same or higher SV leads to increased CO. Blood flow through the coronary arteries increases when the heart works harder.

Which of the following is NOT a function of the three most abundant plasma proteins?

Carry oxygen Correct! Hemoglobin is the protein responsible for oxygen transport and it is found almost entirely in RBCs, not dissolved in the plasma. The three most abundant plamsa proteins are albumins, globulins & fibrinogen. Albumins are important for carrying fat soluble substances & maintaining osmotic pressure of the blood. Globulins can be clotting factors, transport proteins or function during immunity (as antibodies). Fibrinogen is a dissolved protein which, when precipitated, forms a fibrin mesh to stop blood loss.

What factors determine blood flow through a vessel? (Assume no factor is 0)

Change in pressure from the proximal to the distal end of the vessel and resistance to blood flow in the vessel

This graph is Model 2B from Worksheet 2. According to the graph, at a change in pressure of 60 mmHg, what is the approximate blood flow for each of the vessels?

Constricted = ~.75 ml/min Normal = ~1.5 ml/min Dilated = ~5.5 ml/min

This graph is Model 2B from Worksheet 2. According to the graph, at what change in pressure do all vessels have a blood flow of 2 ml/min?

Constricted = ~110 mmHg Normal = ~70 mmHg Dilated = ~25 mmHg

During the life (or death) of erythrocytes:

Developing RBC's contain nuclei and ribosomes

When is pressure in both ventricles high enough to close the AV valves, but too low to open the semilunar valves?

During the earliest phases of ventricular systole and diastole Right! When both valves (AV and semi-lunar) are closed, the ventricles can neither fill nor empty. This occurs at two times during the cardiac cycle. The first is when the ventricles begin contracting; the pressure in the ventricles exceeds that in the atria, causing the AV valves to close (with the AV valves closed, the ventricles cannot fill). But, the pressure in the ventricles is still lower than that in the pulmonary trunk and aorta and so the blood cannot flow out of the heart into these arteries. This phase is called isovolumetric contraction - because the volume is the same in the ventricles during this time period, nothing in, nothing out. All valves are closed again during the initial phase of ventricular diastole called isovolumetric relaxation. Again, this terminology refers to the volume in the ventricles, nothing in, nothing out. Isovolumetric relaxation occurs after ventricular ejection, but before ventricular filling. The pressure in the ventricles is lower than the great vessels, and backward flowing blood causes the SL valves to close (preventing complete backward flow into the ventricles). But pressure in the ventricles is still higher than pressure in the atria, and as such, the AV valves are still closed. I think of an elevator being used to move someone to a 6th floor apartment. When you load the elevator at the ground floor (elevator filling), the elevator is at a very low building level and the door in the apartment building lobby is open. Once the elevator car is full, you close the door and go up. As the elevator climbs the elevator shaft to the 6th floor, nothing can get in the elevator or out (lobby door closed, 6th floor door closed). Once you reach the 6th floor, you open the 6th floor door and unload (eject). When the elevator is empty, the 6th floor door closes and the elevator drops back down again - but nothing can get in or out as the elevator travels back down to the lobby again.

On Model 2, when does blood eject from the ventricles out into the aorta?

During time 4

When is pressure in the ventricle highest?

During ventricular ejection Right! Blood flow is determined by pressure gradients and blood flows from high pressure to low pressure. During a certain phase of ventricular systole (contraction), blood is ejected out of the ventricles into the arteries leaving the heart. Blood leaves the ventricles because the pressure in the ventricle exceeds the pressure in the arteries.

What does a blood pressure of 120/70 tell you?

During ventricular systole, the left ventricle generates more than 120 mmHg Right! Blood must flow from high pressure to low. A blood pressure of 120/70 means that during ventricular systole, when blood is leaving the ventricles and distending the aorta and elastic systemic arteries, the blood pressure in the measured artery is 120 mmHg. During ventricular diastole, there is no further ejection and the elastic arteries recoil. The arteries recoil, keeping pressure on the blood as its volume decreases (the blood is moving away from the SL valve into the systemic circuit). The diastolic pressure is the lowest pressure recorded in the arteries and it occurs just before the next ejection begins and more blood is ejected into the aorta, distending it again (here, 70 mmHg). In order to initiate ventricular ejection into the aorta, the left ventricle must at first overcome 70 mmHg, but as volume continues to flow into the aorta, the pressure rises up to 120 mmHg. At peak systole, to continue blood flow into the aorta, ventricular pressure must exceed 120 mmHg. In this way diastolic pressure represents the pressure that must first be overcome to begin ejection and systolic pressure represents the peak pressures required to sustain ejection at the height of systole.

A baby is born with blood plasma proteins that attack A antigens (born with anti-A antibodies). What blood type must this baby be?

Either Type B or Type O Yes! The role of plasma antibodies is to attack specific cells bearing certain antigens. Pathogens display antigens that are different from your normal cell antigens. If your antibodies see an antigen that is not from your body, they attack it and destroy the cell bearing those foreign antigens. In addition to your DNA determining the type of antigens on your RBCs, your DNA also determines if you have certain antibodies in your blood plasma. Whatever antigens your RBCs display (for A & B), your body will automatically make antibodies against whatever your RBCs don't display. For example: if all your RBCs have only the A antigen (type A or AB blood), you will never form antibodies against A. If your RBCs never display B antigens (type A or O blood), you will form antibodies against B so that if your immune system ever sees a B antigen, it will attack and destroy. In this question I tell you that a baby has blood plasma antibodies against A antigens - therefore, that baby must never naturally produce A antigens on its RBCs or else it would attack its own blood all the time. So this baby must either be type B or type O.

Large arteries near the heart passively expand and recoil as blood is pumped into them from the ventricles. Which of the following is the classification of these large arteries?

Elastic arteries

Using the blood typing chart above, who's blood contains Anti-A antibodies?

Erik & Ben nonag antiA

Using the blood typing chart above, who has the B antigen on their red blood cells?

Erik & vanessa bc coag antiB attacks B

True or False? The myocardium receives its oxygen supply from the endocardium via diffusion through gap junctions.

False

True or False? The left side of the heart pumps more blood than the right side of the heart (in volume).

False Right! Because the CV system is a closed system, whatever volume leaves the heart ultimately returns. Therefore, both sides of the heart must pump the same volume at the same time to keep flow the same through either side of the heart. If both sides do not pump the same volume whichever ventricle pumps less will have excess blood remaining in the chamber. Then excess blood will back up into the atrium and eventually into the circuit (pulmonary or systemic). If too much blood stays in pulmonary capillaries, the lungs fill with fluid and you cannot load oxygen into your blood - you would feel like you were suffocating. If too much blood remains in the systemic capillaries, the tissues fill with fluid and your limbs become swollen. This condition of unmatched ventricular pumping is called congestive heart failure

True or False? The right ventricle has a thicker myocardium than the left ventricle and it contracts and relaxes before the left ventricle.

False Right! No part of this sentence is true. The left ventricle has a thicker myocardium because it pumps to the entire systemic circuit and must overcome more resistance than the pulmonary circuit. Also, both ventricles contract and relax at the same time so that the same amount of blood is ejected into both circuits at the same time.

Which of the following is essential to the proper coordination of a single heart beat?

Gap junctions between adjacent cardiac muscle cells Right! The cells of the intrinsic conduction system are cardiac muscle cells that do not contract. Instead, they are responsible for automatically depolarizing to fire action potentials that control the contraction cycle of the heart. Action potentials spread through the heart via the gap junctions in the intercalated discs. An action potential is initiated in the SA node. Ions spread from the SA node cells to the atrial muscle via gap junctions - these arriving ions initiate action potentials in the contractile cells. The signal travels to the AV node, AV bundle, bundle branches, Purkinje fibers and finally reaches the ventricular muscle cells. It is gap junctions between all these cells that spreads the signal - and it is ions (sodium, calcium) that actually move through the gap junctions to trigger the next action potential. Even the intrinsic conduction system cells (which are non-contractile cardiac muscle cells) are connected to one another and the contractile cells by gap junctions.

Which of the following is essential to the proper coordination of a single heart beat?

Gap junctions between adjacent cardiac muscle cells Right! The cells of the intrinsic conduction system are cardiac muscle cells that do not contract. Instead, they are responsible for automatically depolarizing to fire action potentials that control the contraction cycle of the heart. Action potentials spread through the heart via the gap junctions in the intercalated discs. An action potential is initiated in the SA node. Ions spread from the SA node cells to the atrial muscle via gap junctions - these arriving ions initiate action potentials in the contractile cells. The signal travels to the AV node, AV bundle, bundle branches, Purkinje fibers and finally reaches the ventricular muscle cells. It is gap junctions between all these cells that spreads the signal - and it is ions (sodium, calcium) that actually move through the gap junctions to trigger the next action potential. Even the intrinsic conduction system cells (which are non-contractile cardiac muscle cells) are connected to one another and the contractile cells by gap junctions.

Donated plasma is often separated into its many useful components. What plasma component would be useful in immunodeficient individuals without antibodies?

Globulins

Ian visits the doctor because he is feeling weak and tired. His hematocrit is 50% and his hemoglobin concentration is 10 g/dL of blood. Which of the following most likely explain his symptoms?

He has a low ability to transport oxygen throughout his body Right! With the values given, the hematocrit is high, but still within normal ranges but his hemoglobin is low. Hemoglobin within the RBCs binds oxygen, allowing the blood to be 70X more able to carry oxygen than when hemoglobin is not present. To make hemoglobin, the body needs iron and certain amino acids. Without enough iron, red blood cells cannot make hemoglobin. When the body senses that oxygen delivery is low (anemia), it signals more red blood cells to be produced to increase oxygen delivery. However, if the new red blood cells still cannot make hemoglobin, the cycle continues causing high RBC counts, but low hemoglobin counts. Typically, hemoglobin levels (by number) should be 1/3rd the hematocrit. So, for a hematocrit of 50, you would expect hemoglobin levels of about 16-17 g/dL. The place in the body that senses oxygen carrying capacity of the blood is the kidney (mostly). When low oxygen levels are detected, cells of the kidney release a hormone called erythropoietin (EPO). EPO travels to the bone marrow and stimulates RBC and hemoglobin productions. In the bone marrow, stem cells called hemocytoblasts produce all formed elements are derived (formed elements are the RBCs, WBCs and platelets). If formed elements were not present, the hematocrit would not be within normal ranges. Beyond other things needed for cell production, vitamin B12 is required for production of RBCs. Without vitamin B12, hematocrit would be very low. The condition pernicious anemia develops when vitamin B12 is not able to absorbed by the body (usually due to gastric ulcers/disease). There is no mention of platelet counts. However, capillaries can clog with RBCs when the hematocrit is very high (polycythemia). This condition is not likely to be observed here. Given that the hematocrit is within normal limits, it is unlikely that the RBC number is low.

erythropoiesis.png The image above shows erythropoiesis, the process of red blood cell formation. Erythrocytes (RBCs) differentiate from pleuripotent stem cells called hemocytoblasts. This process take about 5-7 days. Using the Erythropoiesis image, which is true comparing hemocytoblasts and erythrocytes?

Hemocytoblasts are larger than erythrocytes

What intracellular blood protein transports oxygen and carbon dioxide?

Hemoglobin Right! Hemoglobin is the protein found in red blood cells. Its job is to transport respiratory gases, oxygen and carbon dioxide. Oxygen binds to hemoglobin as the red blood cells move through the pulmonary (lung) capillaries; it unbinds from hemoglobin when the red blood cell enters the systemic (peripheral) capillaries allowing the oxygen to move to the tissue cell mitochondria. Carbon dioxide is produced in the mitochondria of the peripheral tissues. It binds to hemoglobin (on a different part than where the oxygen binds) and is carried away from the tissues; it detaches from hemoglobin in the pulmonary capillaries where it can move into the air of the lungs and be breathed out. Hemoglobin is made in the red bone marrow where red blood cells are also made. Albumin, globulin and fibrinogen are plasma proteins which function not to carry respiratory gases. Albumins are important to maintaining blood osmotic pressure and transporting substances in the blood. Globulins are important for immunity (and some transport of hormones). Fibrinogen is important for blood clotting. Most plasma proteins are made in the liver.

How do you feel about this statement? Blood returns from lungs via pulmonary arteries to the left atrium.

I think it should say: Blood returns from lungs via pulmonary veins to the left atrium.

Using the Blood O2 image above, what would be the effect on hematocrit of injecting oneself with synthetic erythropoietin?

Increased hematocrit

Regarding cardiac output:

Increased venous return increases stroke volume and cardiac output Right! Increased heart rate will always lower cardiac output because the ventricles fill less. - Not true. Although EDV may decrease, if you increase contractility, the ventricle will squeeze harder and eject more, lowering the amount left behind (ESV). This may keep SV normal or even increased, thus maintaining or increasing CO. Furthermore, increased exercise may increase venous return, so that even though filling time is less, more volume may come back, maintaining EDV. People with slow heart rates always have low cardiac outputs. - Not true. Slower heart rate allows more filling time and a higher EDV. If EDV is higher, SV will be higher due to the Starling law of the heart. Increased venous return increases stroke volume and cardiac output. True. This is the Starling law of the heart. When more volume comes into the ventricle, that volume stretches the muscle into better interaction between actin and myosin. This better interaction means a more forceful contraction. This more forceful contraction leads to more volume being ejected (a higher SV) which means a higher CO. Increasing heart rate will always increase cardiac output. - Not true. If HR is high, but SV is low, CO will not increase. CO = HR x SV. This type of thing may happen when someone loses a lot of blood (hemorrhage). The heart rate increases, but with a low stroke volume (due to low blood volume) they may not be able to maintain CO needed to maintain life.

According to our worksheet, what leaves the capillary through intercellular clefts and enters the interstitial fluid?

Ions, glucose, water

For a single cardiac cycle, why does sympathetic activation increase stroke volume?

It decreases ESV Right!Sympathetic activation increases contractility of the heart, making ventricular systole more effective. This lowers the volume left in the ventricle at the end of a contraction (ESV). Since stroke volume is the difference between how full the ventricle is before it contracts and how empty it is at the end of a contraction, sympathetic input increases stroke volume.

Using the blood typing figure above, suppose Lindsay needed a substantial plasma donation. Who would be the best person to donate large amounts of plasma to Lindsay?

Jason has no antibodies so can donate mass amounts to lindsay w A+

Who has the A antigen on their red blood cells?

Jason & Lindsay w agglutinated anti A

What blood products commonly leave capillaries, enter the interstitial space and migrate into the lymphatic vessels?

Leukocytes

Lindsay needs a blood transfusion. Which of her family members should donate RBCs to Bilal?

Lindsay would have a transfusion reaction to RBCs from all three of her family members none have only A antigen and - must go to +

Which of the following is most likely in individuals with kidney failure?

Low hematocrit Right! The hormone that regulates red blood cell production (erythropoietin, EPO) is produced by the kidney. When the kidney detects low blood oxygen, it releases EPO stimulating red blood cell formation (erythropoiesis). In renal (kidney) failure, the kidney cells do not work properly and thus the level of blood oxygen cannot be detected and EPO cannot be produced. Consequently, these individuals frequently have low red blood cell count (low hematocrit). White blood cell production is regulated by hormones from other white blood cells. Platelet production is regulated by a hormone from the liver. Most clotting factors are produced by the liver and the kidney is not important in their production.

What happens if capillary colloid osmotic pressure is greater than capillary hydrostatic pressure? (Imagine interstitial hydrostatic pressure and interstitial osmotic pressure are both 0 mmHg.)

More fluid would be pulled into the capillary from the interstitial fluid than would be pushed out from the capillary plasma to the interstitial fluid.

Which of the following would you expect to have the thickest tunica media relative to their overall diameter?

Muscular arteries Right! Muscular arteries have large amounts of smooth muscle in their tunica media. Sympathetic nerves innervate this muscle and when active determine the vessel diameter and thus blood pressure and blood flow. They are important in maintaining vascular tone.

Using the Blood O2 image above, and using what you know about hematopoiesis, what cells in the red bone marrow do you think are targeted by erythropoietin? Hint: you will likely need to review the Lecture 2 slides for this one!

Myeloid stem cell

Which of the following do NOT activate cells of the healthy heart?

Neurotransmitter arriving at contractile cells

Lindsay is a mother who gave birth to Maggie (first) and then James (second). Did Lindsay require Rhogam, a therapeutic medication used to prevent the formation of anti-Rh antibodies, during her pregnancies?

No, Lindsay cannot make anti-Rh antibodies she is Rh+ so anti Rh antibodies can NEVER be made

As presented in the capillary shown in Model 3 (be sure to use your data table), is all the fluid that is pushed into the interstitial space from the arterial end reabsorbed at the venous end? If not, where does it go?

No, the extra fluid is drained away by the lymph vessels.

Systole is contraction of heart muscle. The atria contract independently of the ventricles. When is atrial systole with respect to the P wave of the ECG?

Part way through the P wave until about the QRS complex

Systole is contraction of heart muscle. The atria contract independently of the ventricles. When is ventricular systole with respect to the QRS wave/complex of the ECG?

Part way through the QRS wave until about the T wave

According to our worksheet 3, what creates the colloid osmotic pressure of the capillary?

Plasma proteins

A patient experiencing shortness of breath due to reduced oxygen carrying capacity of the blood would benefit most from

RBC donation

During the time of early ventricular filling, which has the lowest pressure in the cardiovascular system?

Right ventricle Right! During early ventricular filling, blood is flowing into the ventricle from the atrium. Therefore blood pressure must be lower in the ventricle than the atrium because blood flows from high pressure to low pressure. Pressure in the aorta and pulmonary trunk is definitely higher than either the atrium or the ventricle at this time as well. Pressure in the inferior vena cava is higher than the pressure in the atrium because blood is flowing into the atrium from the vena cavae.

Vivian has some blood work done. Her physician reports that while all else is normal, she has a high eosinophil count. Which of the following is her physician's most likely response?

She likely has a parasitic worm infection Right! Leukocytes proliferate when they are battling infections. Depending on their specialty, a high amount of a particular leukocyte will indicate an infection of a particular nature. Eosinophils are parasitic worm specialist (they also play a role during allergic and asthma reactions). Of the available options, a high eosinophil count most likely indicates the body is responding to a parasitic worm infection. Neutrophils are bacterial specialists. Monocytes and certain lymphocytes are virus specialists. Bone marrow cancers can affect specific cell lineages and lead to overabundance of particular cell types, but it is more common that it occurs higher in the division process and affects multiple cell types. Furthermore, with bone marrow cancers, the cells that are produced are immature and not well differentiated enough to be called eosinophils.

Using the Erythropoiesis image one more time, why does the early erythroblast need to synthesize ribosomes?

The cell needs ribosomes to make hemoglobin

How are the left and right sides of the heart similar or different?

The left ventricle consumes more oxygen than the right ventricle. Right! The left ventricle has more muscle mass and therefore requires more blood to be delivered to it by the coronary circulation. The left side generates much greater pressures than the right side because it must pump to the entire systemic circuit which has a very large resistance. Therefore, the left side must generate more pressure to overcome the resistance and keep blood moving to the tissues. All of the other options are incorrect because they either state or imply that the amount of blood returning to or leaving the heart is unequal on both sides. This is of course incorrect because the left and right sides must move the same amount of blood per unit time (same cardiac output) to prevent blood backing up in a circuit. When the left and right sides are not matched, this is called congestive heart failure.

What determines your blood type?

The proteins on your red blood cell membranes as dictated by your DNA Yes! All cells in your body have antigens (name tags) on their cell membranes that identify them as belonging to your body. Your DNA dictates their formation and expression on the cell membrane surface. Red blood cells are no different in this respect. While there can be many types of antigens that body cells may express, red blood cells regularly express those that fall into about 23 different groups. The most common groups are the ABO group and the Rh group. If all of your RBCs display A antigens and Rh antigens, you are type A+ blood. If they display A, B and Rh antigens, you are type AB+ blood. If your RBCs display neither A nor B and also not Rh, you are O- blood type.

Which of the following best explains why the systemic circuit pressure gradient is so much greater than the pulmonary circuit pressure gradient?

The pulmonary circuit has lower total resistance than the systemic circuit

What would most likely happen if the AV bundle were damaged?

The ventricles would not be activated appropriately after the atria Correct! The heart relies on coordination between the atria and ventricles to an effective pump. The AV bundle is the only electrical connection between the atria and ventricles that ensures the activation of the ventricles only after the atria contract. The SA node activates the atria and the signal spreads through the atria to the AV node. From the AV node, the signal passes through the AV bundle, down the bundle branches to the Purkinje fibers and ultimately, the ventricular muscle mass. If the bundle is damaged, the ventricles will receive signals from other intrinsic conduction cells in the ventricular muscle mass. The bundle branch cells or the Purkinje fibers can fire spontaneously and trigger ventricular contraction. Usually when this happens, normally timed QRS complexes are not seen on the ECG and they seem to occur at their own rate regardless of the appearance of the p wave. This question does not state that the SA node is damaged, so there is no reason to assume atrial non-function.

Red blood cells do not have mitochondria although they do have glycolytic enzymes. What is an advantage of this in these cells?

They do not use the oxygen they carry for ATP production Right! RBCs lack nuclei and mitochondria as well as most other organelles. This allows more room for hemoglobin content in the cell - which then enables more oxygen carrying capacity. However, RBCs do use ATP for some processes and when they need ATP, they make it anaerobically using glycolytic enzymes. The advantage of not using oxygen is that they can deliver it to hungry tissues without reducing the blood's oxygen content before it reaches the tissues.

Using Model 2, when is ventricular systole?

Time period 3 & 4

True or False? If aortic pressure increased above normal values, the left ventricle would need to generate even more pressure to ensure blood flow through the systemic circuit.

True Right! Blood flows from high pressure to low pressure. In order to move blood from the left ventricle to the aorta, the left ventricle pressure must be greater than the aorta. When systemic arterial pressure increases pathologically (due to loss of elasticity or fatty plaque build up), this forces the left ventricle to work even harder to ensure proper blood flow. Everyone must maintain a minimal blood flow from the heart (termed cardiac output) no matter what. Too high aortic pressure will result in less blood ejection (lower SV) which would then decrease cardiac output (CO). Your body would then need to increase contractility or HR to keep CO high enough. This is why increased systemic blood pressure is so dangerous to your health - it increases the workload on the heart and the heart can only handle that problem for so long before it fails. High blood pressure is particularly dangerous for individuals with weakened hearts (such as after a heart attack)

Under normal resting conditions or light exercise, the primary factor altering cardiac output is:

Venous return Right! Venous return is the volume of blood returning to the heart from systemic veins. It is the primary mechanism for changing cardiac output as explained by the Frank-Starling law of the heart. The more blood that flows into the heart, the more blood that will be pumped out thus altering cardiac output. Changes in hormones or neural stimuli do alter cardiac output, but the primary mechanism that acts to alter cardiac output to match tissue demands at rest and during routine activity is venous return. Parasympathetic activation keeps cardiac output low at rest, but it does not affect changes to match needs during shifts in venous return.

Which of the following prevent backflow in the large veins of the lower extremity

Venous valves Right! Just like in the heart, valves in the veins prevent backflow of blood (backflow back down to the feet in the case of the veins). Skeletal muscle contractions also help venous blood move upward toward the heart, as does negative intrathoracic pressure generated during ventilation of the lungs. The vasa vasorum is the blood supply of the vessels - the vessels are tissue too and need their own source of nutrients to live. The blood moving though vessels cannot supply all the vessel tissue with nutrients, just like the blood inside the heart chambers does not supply nutrients to the heart muscle.

What is happening to ventricular volume during times 3 & 5?

Ventricular volume is not changing in 3 or in 5

The liver is responsible for the production of most clotting factors. Which of the following best describes the role of these clotting factors when a blood vessel is broken?

When a vessel ruptures, clotting factors become activated to permit the conversion of soluble fibrinogen to insoluble fibrin. Right! Clotting factors are chemicals or proteins used in the multi-step process that ultimately converts soluble plasma fibrinogen into insoluble fibrin. Fibrinogen floats around in the blood dissolved in the plasma. When vessel injury occurs, platelets stick to the site of injury and release their contents. This causes a temporary platelet plug to form. Next, clotting is initiated. This process causes fibrin strings to form which then span the injury and trap blood cells, thus sealing the break in the vessel. For fibrin to precipitate out of the plasma, the protein (enzyme) thrombin must be formed. Thrombin is found in the blood in its inactivated form, prothrombin (made by liver). To convert prothrombin to thrombin, other clotting factors found in the blood are converted from their inactive forms to active forms. The initial stimulus causing activation of all clotting factors can be either the exposed collagen from a broken vessel or a chemical released from damaged tissue. The chain reaction that occurs ultimately results in clotting.

Can stroke volume increase if EDV does not change?

YES Right! EDV is end diastolic volume (it is how full the ventricle is before it contracts). ESV is end systolic volume - it is how full the ventricle is at the end of a ventricular contraction (not all blood is actually pumped out of the ventricle with each beat - there is always some residual volume). The amount that is ejected is called SV (stroke volume) and it is calculated by this formula: SV = EDV-ESV. So, stroke volume can increase by increasing EDV or decreasing ESV. ESV will go down if the heart squeezes harder and ejects more. The higher the stroke volume, the greater the cardiac output (for a given heart rate).

Neutrophil

bacteria slayer

Eosinophil

combats parasitic worms

Lymphocyte

create antibodies & attack infected/cancerous cells

Using the graph, S:

ensures that the heart does not achieve tetanus. ??

True or False? To increase blood flow to a working muscle, sympathetic activation would constrict the arteriole serving that skeletal muscle.

false

True or False? The left side of the heart pumps more blood than the right side of the heart (in volume).

false Right! Because the CV system is a closed system, whatever volume leaves the heart ultimately returns. Therefore, both sides of the heart must pump the same volume at the same time to keep flow the same through either side of the heart. If both sides do not pump the same volume whichever ventricle pumps less will have excess blood remaining in the chamber. Then excess blood will back up into the atrium and eventually into the circuit (pulmonary or systemic). If too much blood stays in pulmonary capillaries, the lungs fill with fluid and you cannot load oxygen into your blood - you would feel like you were suffocating. If too much blood remains in the systemic capillaries, the tissues fill with fluid and your limbs become swollen. This condition of unmatched ventricular pumping is called congestive heart failure.

True or False? The right ventricle is the pump for the pulmonary circuit.

false but true?? The right ventricle pumps blood to the pulmonary trunk and then the blood goes to the lungs. The left ventricle pumps blood to the aorta and then the blood goes to the systemic circulation.

Which of the following will directly increase cardiac output?

increased EDV with no change in HR Right! CO = HR x SV SV = EDV - ESV Increasing EDV leads to a higher SV because of the Frank-Starling law of the heart. The more blood that returns to the heart, the stronger the heart contraction to empty more blood. Any increase in ESV will decrease CO because of a lower SV. A decreased SA node firing rate is the same as slowing the heart rate, and a lower HR with the same or lower SV means a lower CO.

What surrounds (and is in contact with) muscles, gland, and other body cells, and gives rise to lymph fluid?

interstitial fluid

The image above shows the negative feedback mechanism that maintains blood O2 levels. Using the Blood O2 image above, what organ releases erythropoietin?

kidney

Monocyte

largest WBC, include phagocytic macrophages

Does the left ventricle fill with new blood during all of ventricular diastole? (Look at question 12 of worksheet)

no isovolumetric during 5

Does the ejection phase (ventricles emptying into aorta) occur during all of ventricular systole? (Look at question 12 to help you)

no not 3 when pressure is building up

1 2 3 4 5 on graph

passive fill of ventricles, atrial kick or active fill of ventricles, isovolumetric contraction, ventricular ejection, isovolumetric relaxation

Using the Erythropoiesis image again, what is the name of the cell that is first seen without a nucleus?

reticulocyte

During the time of early ventricular filling, where is the lowest pressure in the cardiovascular system?

right ventricle Right! During early ventricular filling, blood is flowing into the ventricle from the atrium. Therefore blood pressure must be lower in the ventricle than the atrium because blood flows from high pressure to low pressure. Pressure in the aorta and pulmonary trunk is definitely higher than either the atrium or the ventricle at this time as well. Pressure in the inferior vena cava is higher than the pressure in the atrium because blood is flowing into the atrium from the vena cavae.

True or False? The right ventricle is the pump for the pulmonary circuit.

true Right! The right ventricle pumps blood to the pulmonary trunk and then the blood goes to the lungs. The left ventricle pumps blood to the aorta and then the blood goes to the systemic circulation.

True or False? The systemic circuit contains more blood volume than the pulmonary circuit.

true Right! The systemic circuit delivers oxygen rich blood to the tissues and carries away oxygen poor blood. It is much longer and has more capacity than the pulmonary circuit because it has more tissues to serve. Interestingly, blood volume distribution between the arteries, veins and capillaries in the systemic circuit is not equal. At rest 64% of your blood volume is found in the flabby, thin walled systemic circulation veins. These veins are called the volume reserve of the cardiovascular system. During exercise, skeletal muscle contractions squeeze the veins that lie between them and move more of the pooled venous blood toward the heart. Sympathetic activation during intense exercise also increases the pressure in these vessels, encouraging further blood flow back to the heart - this increased flow towards the right side of the heart is termed increased venous return. Blood flow is volume per unit time. Meaning, 2 ml/sec. Even though capacities and total volumes of the systemic and pulmonary circuits are different, blood flow (the amount entering and the amount leaving each circuit) are always equal. Additionally, blood velocity and blood flow are also not equal. Velocity refers to the distance traveled in a length of time, flow is a volume in a length of time. The same volume (say 5L) may enter and leave the heart every minute, but if the pulmonary circuit is short and the systemic circuit is long, the same volume can travel through the short distance very slowly, while it must travel the long distance very quickly.


Conjuntos de estudio relacionados

Week 1/Lecture 1 Prep: Anatomical Planes of the Human Body/Directional Terms

View Set

Virginia Salesperson Real Estate DPOR Quizlet

View Set

Chapter 16: Nursing Management During the Postpartum Period

View Set

Macroeconomics FINAL FINAL FINAL

View Set

Anatomy 10A Final Study Guide: Unit 3&4

View Set

Pharmacology Exam #2 Chapters 22, 23, 24

View Set

日本語総まとめN2語彙 第1週 1日目

View Set